www.vorhilfe.de
Vorhilfe

Kostenlose Kommunikationsplattform für gegenseitige Hilfestellungen.
Hallo Gast!einloggen | registrieren ]
Startseite · Forum · Wissen · Kurse · Mitglieder · Team · Impressum
Forenbaum
^ Forenbaum
Status Vorhilfe
  Status Geisteswiss.
    Status Erdkunde
    Status Geschichte
    Status Jura
    Status Musik/Kunst
    Status Pädagogik
    Status Philosophie
    Status Politik/Wirtschaft
    Status Psychologie
    Status Religion
    Status Sozialwissenschaften
  Status Informatik
    Status Schule
    Status Hochschule
    Status Info-Training
    Status Wettbewerbe
    Status Praxis
    Status Internes IR
  Status Ingenieurwiss.
    Status Bauingenieurwesen
    Status Elektrotechnik
    Status Maschinenbau
    Status Materialwissenschaft
    Status Regelungstechnik
    Status Signaltheorie
    Status Sonstiges
    Status Technik
  Status Mathe
    Status Schulmathe
    Status Hochschulmathe
    Status Mathe-Vorkurse
    Status Mathe-Software
  Status Naturwiss.
    Status Astronomie
    Status Biologie
    Status Chemie
    Status Geowissenschaften
    Status Medizin
    Status Physik
    Status Sport
  Status Sonstiges / Diverses
  Status Sprachen
    Status Deutsch
    Status Englisch
    Status Französisch
    Status Griechisch
    Status Latein
    Status Russisch
    Status Spanisch
    Status Vorkurse
    Status Sonstiges (Sprachen)
  Status Neuerdings
  Status Internes VH
    Status Café VH
    Status Verbesserungen
    Status Benutzerbetreuung
    Status Plenum
    Status Datenbank-Forum
    Status Test-Forum
    Status Fragwürdige Inhalte
    Status VH e.V.

Gezeigt werden alle Foren bis zur Tiefe 2

Navigation
 Startseite...
 Neuerdings beta neu
 Forum...
 vorwissen...
 vorkurse...
 Werkzeuge...
 Nachhilfevermittlung beta...
 Online-Spiele beta
 Suchen
 Verein...
 Impressum
Das Projekt
Server und Internetanbindung werden durch Spenden finanziert.
Organisiert wird das Projekt von unserem Koordinatorenteam.
Hunderte Mitglieder helfen ehrenamtlich in unseren moderierten Foren.
Anbieter der Seite ist der gemeinnützige Verein "Vorhilfe.de e.V.".
Partnerseiten
Dt. Schulen im Ausland: Mathe-Seiten:

Open Source FunktionenplotterFunkyPlot: Kostenloser und quelloffener Funktionenplotter für Linux und andere Betriebssysteme
Forum "Uni-Komplexe Analysis" - Grad von Polynom
Grad von Polynom < komplex < Analysis < Hochschule < Mathe < Vorhilfe
Ansicht: [ geschachtelt ] | ^ Forum "Uni-Komplexe Analysis"  | ^^ Alle Foren  | ^ Forenbaum  | Materialien

Grad von Polynom: Frage (beantwortet)
Status: (Frage) beantwortet Status 
Datum: 18:02 Di 14.06.2016
Autor: Herzblatt

Aufgabe
a) Es seien [mm] A,B,\alpha [/mm] >0. Sei f: [mm] \IC\to \IC [/mm] ganz und es gelte |f(z)| [mm] \le A+B|z|^\alpha [/mm] für alle z [mm] \in \IC. [/mm] Zeigen Sie, dass f ein Polynom ist, dessen Grad höchstens [a] beträgt.
b) Bestimmen Sie alle holomorphen Funktionen g: [mm] \IC \setminus [/mm] {0} [mm] \to \IC [/mm] mit der Eigenschaft
[mm] |g(z)|\le \sqrt{|z|}+\frac{1}{\sqrt{|z|}} [/mm]

Hallo :-)

zu Aufgabe a)
ich weiß, dass f holomorph und beschränkt ist, aber was genau ist A und B? sind das Elemente aus [mm] \IC [/mm] oder Funktionen? Die Klammern in denen a steht sind übrigens oben nicht geschlossen (dieses Zeichen gab es hier nur nicht ). Was bedeutet dieses Zeichen dann?

zu Aufgabe b)
kann mir jemand einen Tipp geben? Kann man hier was mit Potenzeihen machen?Holomorphe FUnktionen lassen sich ja schliesslich in eine Potenzreihe entwickeln....


Vielen Dank :-)

        
Bezug
Grad von Polynom: Antwort
Status: (Antwort) fertig Status 
Datum: 21:02 Di 14.06.2016
Autor: Gonozal_IX

Hiho,

>  ich weiß, dass f holomorph und beschränkt ist, aber was
> genau ist A und B? sind das Elemente aus [mm]\IC[/mm] oder
> Funktionen?

Ich zitiere:

> a) Es seien [mm]A,B,\alpha[/mm] >0

Damit sind [mm] $A,B,\alpha$ [/mm] insbesondere Elemente aus [mm] $\IR$. [/mm]

> Die Klammern in denen a steht

1.) Was ist a? Ich vermute du meinst [mm] $\alpha$ [/mm]

> sind übrigens oben nicht geschlossen (dieses Zeichen gab es hier nur nicht ).

Doch: \lfloor und \rfloor und zusammen ergibt das dann [mm] $\lfloor\alpha\rfloor$ [/mm]

>  Was bedeutet dieses Zeichen dann?

Die größte ganzzahlige Zahl, die kleiner ist als [mm] $\alpha$ [/mm] oder lapidar gesagt: Runde [mm] $\alpha$ [/mm] ab.

> zu Aufgabe b)
>  kann mir jemand einen Tipp geben? Kann man hier was mit
> Potenzeihen machen?Holomorphe FUnktionen lassen sich ja
> schliesslich in eine Potenzreihe entwickeln....

Multipliziere mit [mm] $\sqrt{|z|}$ [/mm] und wende a) an.

Gruß,
Gono

Bezug
        
Bezug
Grad von Polynom: Antwort
Status: (Antwort) fertig Status 
Datum: 08:46 Mi 15.06.2016
Autor: fred97


> a) Es seien [mm]A,B,\alpha[/mm] >0. Sei f: [mm]\IC\to \IC[/mm] ganz und es
> gelte |f(z)| [mm]\le A+B|z|^\alpha[/mm] für alle z [mm]\in \IC.[/mm] Zeigen
> Sie, dass f ein Polynom ist, dessen Grad höchstens [a]
> beträgt.
> b) Bestimmen Sie alle holomorphen Funktionen g: [mm]\IC \setminus[/mm]
> {0} [mm]\to \IC[/mm] mit der Eigenschaft
>  [mm]|g(z)|\le \sqrt{|z|}+\frac{1}{\sqrt{|z|}}[/mm]
>  Hallo :-)
>  
> zu Aufgabe a)
>  ich weiß, dass f holomorph und beschränkt ist,

Hmmm...., da weißt Du etwas, das gar nicht stimmt !

Nimm mal f(z)=z

f ist holomorph auf [mm] \IC, [/mm] das stimmt, aber beschränkt .... ?




> aber was
> genau ist A und B? sind das Elemente aus [mm]\IC[/mm] oder
> Funktionen? Die Klammern in denen a steht sind übrigens
> oben nicht geschlossen (dieses Zeichen gab es hier nur
> nicht ). Was bedeutet dieses Zeichen dann?

Das hat Gono Dir gesagt.


>  
> zu Aufgabe b)
>  kann mir jemand einen Tipp geben? Kann man hier was mit
> Potenzeihen machen?Holomorphe FUnktionen lassen sich ja
> schliesslich in eine Potenzreihe entwickeln....

Die Aufgabe hat es in sich...

Der Vorschlag von Gono ist in meinen Augen nicht brauchbar, denn multiplizieren wir mit [mm] \wurzel{|z|} [/mm] durch, so bekommen wir

   [mm] \wurzel{|z|}|g(z)| \le [/mm] 1+|z|  für z [mm] \ne [/mm] 0.

Wenn wir nun a) anwenden wollen, brauchen wir eine ganze Funktion f. Aber welch soll das sein ? Es bietet sich natürlich  [mm] \wurzel{z}g(z) [/mm] an. Das macht aber Probleme, denn g ist in 0 nicht definiert und welche (holomorphe) Wurzel nehmen wir .. ?


Wir multiplizieren mit z durch und bekommen, mit h(z):=zg(z):

(*) $ |h(z)| [mm] \le [/mm] z [mm] \wurzel{|z|}+ \wurzel{|z|}$ [/mm]  für z [mm] \ne [/mm] 0.

Edit: es lautet natürlich so:

   (*) $ |h(z)| [mm] \le [/mm] |z| [mm] \wurzel{|z|}+ \wurzel{|z|}$ [/mm]  für z [mm] \ne [/mm] 0.

h hat in 0 eine isolierte Singularität. Zeige Du, mit (*), dass diese Singularität hebbar ist.

h kann also durch h(0):=0 zu einer ganzen Funktion fortgesetzt werden.

Für z [mm] \ne [/mm] 0 ist

  [mm] g(z)=\bruch{h(z)}{z}=\bruch{h(z)-h(0)}{z-0} \to [/mm] h'(0)   für z [mm] \to [/mm] 0.

g hat in 0 eine isolierte Singularität. Zeige Du, dass diese Singularität hebbar ist.

g kann also durch g(0):=h'(0) zu einer ganzen Funktion fortgesetzt werden.

g ist dann auf [mm] \{z \in \IC: |z| \le 1 \} [/mm] beschränkt. Es gibt also ein c>0 mit

    |g(z)| [mm] \le [/mm] c für |z| [mm] \le [/mm] 1,

also auch

     |g(z)| [mm] \le [/mm] c +|z|  für |z| [mm] \le [/mm] 1.

Für |z| [mm] \ge [/mm] 1 folgt aus  $ [mm] |g(z)|\le \sqrt{|z|}+\frac{1}{\sqrt{|z|}} [/mm] $:

     |g(z)| [mm] \le 1+\sqrt{|z|} \le [/mm] 1+|z|.

Setzt man A:= [mm] \max\{1,c\}, [/mm] so folgt

     |g(z)| [mm] \le [/mm] A+|z|  für alle z [mm] \in \IC. [/mm]

Jetzt können wir a) anwenden und sehen: es gibt a,b [mm] \in \IC [/mm] mit:

     g(z)=a+bz.

Jetzt müssen wir noch genaueres über a und b rauskitzeln.


Aus

     $ [mm] |a+bz|\le \sqrt{|z|}+\frac{1}{\sqrt{|z|}} [/mm] $  für z [mm] \ne [/mm] 0

folgt

       $ [mm] |a+bz^2|\le |z|+\frac{1}{|z|} [/mm] $  für z [mm] \ne [/mm] 0

Das liefert

      [mm] $|az+bz^3| \le 1+|z|^2$ [/mm]  für z [mm] \ne [/mm] 0.

Mit z [mm] \to [/mm] 0 sieht man dann

       [mm] $|az+bz^3| \le 1+|z|^2$ [/mm]  für alle z .

Aufgabenteil a) liefert nun b=0. Das hat zur Folge

   $ [mm] |a|\le \sqrt{|z|}+\frac{1}{\sqrt{|z|}} [/mm] $  für z [mm] \ne [/mm] 0

oder

  $|a| [mm] \le \bruch{1}{r}+r$ [/mm]  für alle r>0

Überlege Dir: [mm] \min\{ \bruch{1}{r}+r: r>0\}=2. [/mm]

Welche Funktionen g sind nun die gesuchten ?

FRED


>  
>
> Vielen Dank :-)


Bezug
                
Bezug
Grad von Polynom: Frage (beantwortet)
Status: (Frage) beantwortet Status 
Datum: 09:48 Do 16.06.2016
Autor: Herzblatt


> > a) Es seien [mm]A,B,\alpha[/mm] >0. Sei f: [mm]\IC\to \IC[/mm] ganz und es
> > gelte |f(z)| [mm]\le A+B|z|^\alpha[/mm] für alle z [mm]\in \IC.[/mm] Zeigen
> > Sie, dass f ein Polynom ist, dessen Grad höchstens [a]
> > beträgt.
> > b) Bestimmen Sie alle holomorphen Funktionen g: [mm]\IC \setminus[/mm]
> > {0} [mm]\to \IC[/mm] mit der Eigenschaft
>  >  [mm]|g(z)|\le \sqrt{|z|}+\frac{1}{\sqrt{|z|}}[/mm]
>  >  Hallo
> :-)
>  >  
> > zu Aufgabe a)
>  >  ich weiß, dass f holomorph und beschränkt ist,
>
> Hmmm...., da weißt Du etwas, das gar nicht stimmt !
>
> Nimm mal f(z)=z
>  
> f ist holomorph auf [mm]\IC,[/mm] das stimmt, aber beschränkt ....
> ?
>  
>
>
>
> > aber was
> > genau ist A und B? sind das Elemente aus [mm]\IC[/mm] oder
> > Funktionen? Die Klammern in denen a steht sind übrigens
> > oben nicht geschlossen (dieses Zeichen gab es hier nur
> > nicht ). Was bedeutet dieses Zeichen dann?
>  
> Das hat Gono Dir gesagt.
>  
>
> >  

> > zu Aufgabe b)
>  >  kann mir jemand einen Tipp geben? Kann man hier was mit
> > Potenzeihen machen?Holomorphe FUnktionen lassen sich ja
> > schliesslich in eine Potenzreihe entwickeln....
>  
> Die Aufgabe hat es in sich...
>  
> Der Vorschlag von Gono ist in meinen Augen nicht brauchbar,
> denn multiplizieren wir mit [mm]\wurzel{|z|}[/mm] durch, so bekommen
> wir
>  
> [mm]\wurzel{|z|}|g(z)| \le[/mm] 1+|z|  für z [mm]\ne[/mm] 0.
>  
> Wenn wir nun a) anwenden wollen, brauchen wir eine ganze
> Funktion f. Aber welch soll das sein ? Es bietet sich
> natürlich  [mm]\wurzel{z}g(z)[/mm] an. Das macht aber Probleme,
> denn g ist in 0 nicht definiert und welche (holomorphe)
> Wurzel nehmen wir .. ?
>  
>
> Wir multiplizieren mit z durch und bekommen, mit
> h(z):=zg(z):
>  
> (*) [mm]|h(z)| \le z \wurzel{|z|}+ \wurzel{|z|}[/mm]  für z [mm]\ne[/mm] 0.

Warum?
Ich erhalte da für
[mm] |h(z)|\le\wurzel{|z|}*z+\frac{z}{\wurzel{|z|}} [/mm]

>  
> h hat in 0 eine isolierte Singularität. Zeige Du, mit (*),
> dass diese Singularität hebbar ist.

Ok ich setzt mich gleich ran ;-)

>  
> h kann also durch h(0):=0 zu einer ganzen Funktion
> fortgesetzt werden.
>  
> Für z [mm]\ne[/mm] 0 ist
>
> [mm]g(z)=\bruch{h(z)}{z}=\bruch{h(z)-h(0)}{z-0} \to[/mm] h'(0)  
> für z [mm]\to[/mm] 0.
>  
> g hat in 0 eine isolierte Singularität. Zeige Du, dass
> diese Singularität hebbar ist.
>  
> g kann also durch g(0):=h'(0) zu einer ganzen Funktion
> fortgesetzt werden.
>  
> g ist dann auf [mm]\{z \in \IC: |z| \le 1 \}[/mm] beschränkt. Es
> gibt also ein c>0 mit
>  
> |g(z)| [mm]\le[/mm] c für |z| [mm]\le[/mm] 1,
>  
> also auch
>  
> |g(z)| [mm]\le[/mm] c +|z|  für |z| [mm]\le[/mm] 1.
>  
> Für |z| [mm]\ge[/mm] 1 folgt aus  [mm]|g(z)|\le \sqrt{|z|}+\frac{1}{\sqrt{|z|}} [/mm]:
>  
> |g(z)| [mm]\le 1+\sqrt{|z|} \le[/mm] 1+|z|.
>  
> Setzt man A:= [mm]\max\{1,c\},[/mm] so folgt
>  
> |g(z)| [mm]\le[/mm] A+|z|  für alle z [mm]\in \IC.[/mm]
>  
> Jetzt können wir a) anwenden und sehen: es gibt a,b [mm]\in \IC[/mm]
> mit:
>  
> g(z)=a+bz.
>  
> Jetzt müssen wir noch genaueres über a und b
> rauskitzeln.
>  
>
> Aus
>
> [mm]|a+bz|\le \sqrt{|z|}+\frac{1}{\sqrt{|z|}}[/mm]  für z [mm]\ne[/mm] 0
>  
> folgt
>  
> [mm]|a+bz^2|\le |z|+\frac{1}{|z|}[/mm]  für z [mm]\ne[/mm] 0
>  
> Das liefert
>  
> [mm]|az+bz^3| \le 1+|z|^2[/mm]  für z [mm]\ne[/mm] 0.
>  
> Mit z [mm]\to[/mm] 0 sieht man dann
>  
> [mm]|az+bz^3| \le 1+|z|^2[/mm]  für alle z .
>  
> Aufgabenteil a) liefert nun b=0.

Das verstehe ich leider noch nicht, warum b=0?

> Das hat zur Folge
>  
> [mm]|a|\le \sqrt{|z|}+\frac{1}{\sqrt{|z|}}[/mm]  für z [mm]\ne[/mm] 0
>  
> oder
>  
> [mm]|a| \le \bruch{1}{r}+r[/mm]  für alle r>0
>  
> Überlege Dir: [mm]\min\{ \bruch{1}{r}+r: r>0\}=2.[/mm]
>  
> Welche Funktionen g sind nun die gesuchten ?
>  
> FRED
>  
>
> >  

> >
> > Vielen Dank :-)
>  


Bezug
                        
Bezug
Grad von Polynom: Antwort
Status: (Antwort) fertig Status 
Datum: 10:15 Do 16.06.2016
Autor: fred97


> > > a) Es seien [mm]A,B,\alpha[/mm] >0. Sei f: [mm]\IC\to \IC[/mm] ganz und es
> > > gelte |f(z)| [mm]\le A+B|z|^\alpha[/mm] für alle z [mm]\in \IC.[/mm] Zeigen
> > > Sie, dass f ein Polynom ist, dessen Grad höchstens [a]
> > > beträgt.
> > > b) Bestimmen Sie alle holomorphen Funktionen g: [mm]\IC \setminus[/mm]
> > > {0} [mm]\to \IC[/mm] mit der Eigenschaft
>  >  >  [mm]|g(z)|\le \sqrt{|z|}+\frac{1}{\sqrt{|z|}}[/mm]
>  >  >  
> Hallo
> > :-)
>  >  >  
> > > zu Aufgabe a)
>  >  >  ich weiß, dass f holomorph und beschränkt ist,
> >
> > Hmmm...., da weißt Du etwas, das gar nicht stimmt !
> >
> > Nimm mal f(z)=z
>  >  
> > f ist holomorph auf [mm]\IC,[/mm] das stimmt, aber beschränkt ....
> > ?
>  >  
> >
> >
> >
> > > aber was
> > > genau ist A und B? sind das Elemente aus [mm]\IC[/mm] oder
> > > Funktionen? Die Klammern in denen a steht sind übrigens
> > > oben nicht geschlossen (dieses Zeichen gab es hier nur
> > > nicht ). Was bedeutet dieses Zeichen dann?
>  >  
> > Das hat Gono Dir gesagt.
>  >  
> >
> > >  

> > > zu Aufgabe b)
>  >  >  kann mir jemand einen Tipp geben? Kann man hier was
> mit
> > > Potenzeihen machen?Holomorphe FUnktionen lassen sich ja
> > > schliesslich in eine Potenzreihe entwickeln....
>  >  
> > Die Aufgabe hat es in sich...
>  >  
> > Der Vorschlag von Gono ist in meinen Augen nicht brauchbar,
> > denn multiplizieren wir mit [mm]\wurzel{|z|}[/mm] durch, so bekommen
> > wir
>  >  
> > [mm]\wurzel{|z|}|g(z)| \le[/mm] 1+|z|  für z [mm]\ne[/mm] 0.
>  >  
> > Wenn wir nun a) anwenden wollen, brauchen wir eine ganze
> > Funktion f. Aber welch soll das sein ? Es bietet sich
> > natürlich  [mm]\wurzel{z}g(z)[/mm] an. Das macht aber Probleme,
> > denn g ist in 0 nicht definiert und welche (holomorphe)
> > Wurzel nehmen wir .. ?
>  >  
> >
> > Wir multiplizieren mit z durch und bekommen, mit
> > h(z):=zg(z):
>  >  
> > (*) [mm]|h(z)| \le z \wurzel{|z|}+ \wurzel{|z|}[/mm]  für z [mm]\ne[/mm] 0.


Ich hatte mich oben verschrieben. Es lautet natürlich

(*) $ |h(z)| [mm] \le [/mm] |z| [mm] \wurzel{|z|}+ \wurzel{|z|}$ [/mm]  für z [mm] \ne [/mm] 0.



>  
> Warum?
> Ich erhalte da für
>  [mm]|h(z)|\le\wurzel{|z|}*z+\frac{z}{\wurzel{|z|}}[/mm]


Es soll so lauten (Beträge !)

[mm]|h(z)|\le\wurzel{|z|}*|z|+\frac{|z|}{\wurzel{|z|}}[/mm]

Weiter ist [mm] \frac{|z|}{\wurzel{|z|}}=\wurzel{|z|} [/mm]



>  >  
> > h hat in 0 eine isolierte Singularität. Zeige Du, mit (*),
> > dass diese Singularität hebbar ist.
>  Ok ich setzt mich gleich ran ;-)
>  >  
> > h kann also durch h(0):=0 zu einer ganzen Funktion
> > fortgesetzt werden.
>  >  
> > Für z [mm]\ne[/mm] 0 ist
> >
> > [mm]g(z)=\bruch{h(z)}{z}=\bruch{h(z)-h(0)}{z-0} \to[/mm] h'(0)  
> > für z [mm]\to[/mm] 0.
>  >  
> > g hat in 0 eine isolierte Singularität. Zeige Du, dass
> > diese Singularität hebbar ist.
>  >  
> > g kann also durch g(0):=h'(0) zu einer ganzen Funktion
> > fortgesetzt werden.
>  >  
> > g ist dann auf [mm]\{z \in \IC: |z| \le 1 \}[/mm] beschränkt. Es
> > gibt also ein c>0 mit
>  >  
> > |g(z)| [mm]\le[/mm] c für |z| [mm]\le[/mm] 1,
>  >  
> > also auch
>  >  
> > |g(z)| [mm]\le[/mm] c +|z|  für |z| [mm]\le[/mm] 1.
>  >  
> > Für |z| [mm]\ge[/mm] 1 folgt aus  [mm]|g(z)|\le \sqrt{|z|}+\frac{1}{\sqrt{|z|}} [/mm]:
>  
> >  

> > |g(z)| [mm]\le 1+\sqrt{|z|} \le[/mm] 1+|z|.
>  >  
> > Setzt man A:= [mm]\max\{1,c\},[/mm] so folgt
>  >  
> > |g(z)| [mm]\le[/mm] A+|z|  für alle z [mm]\in \IC.[/mm]
>  >  
> > Jetzt können wir a) anwenden und sehen: es gibt a,b [mm]\in \IC[/mm]
> > mit:
>  >  
> > g(z)=a+bz.
>  >  
> > Jetzt müssen wir noch genaueres über a und b
> > rauskitzeln.
>  >  
> >
> > Aus
> >
> > [mm]|a+bz|\le \sqrt{|z|}+\frac{1}{\sqrt{|z|}}[/mm]  für z [mm]\ne[/mm] 0
>  >  
> > folgt
>  >  
> > [mm]|a+bz^2|\le |z|+\frac{1}{|z|}[/mm]  für z [mm]\ne[/mm] 0
>  >  
> > Das liefert
>  >  
> > [mm]|az+bz^3| \le 1+|z|^2[/mm]  für z [mm]\ne[/mm] 0.
>  >  
> > Mit z [mm]\to[/mm] 0 sieht man dann
>  >  
> > [mm]|az+bz^3| \le 1+|z|^2[/mm]  für alle z .
>  >  
> > Aufgabenteil a) liefert nun b=0.
>  Das verstehe ich leider noch nicht, warum b=0?


Aus  [mm]|az+bz^3| \le 1+|z|^2[/mm]  für alle z  folgt mit a):

    [mm] az+bz^3 [/mm] ist ein Polynom vom Grad [mm] \le [/mm] 2.

Dann muss b=0 sein

FRED

>  > Das hat zur Folge

>  >  
> > [mm]|a|\le \sqrt{|z|}+\frac{1}{\sqrt{|z|}}[/mm]  für z [mm]\ne[/mm] 0
>  >  
> > oder
>  >  
> > [mm]|a| \le \bruch{1}{r}+r[/mm]  für alle r>0
>  >  
> > Überlege Dir: [mm]\min\{ \bruch{1}{r}+r: r>0\}=2.[/mm]
>  >  
> > Welche Funktionen g sind nun die gesuchten ?
>  >  
> > FRED
>  >  
> >
> > >  

> > >
> > > Vielen Dank :-)
> >  

>  


Bezug
                                
Bezug
Grad von Polynom: Frage (beantwortet)
Status: (Frage) beantwortet Status 
Datum: 18:50 Sa 18.06.2016
Autor: Herzblatt


> > > > a) Es seien [mm]A,B,\alpha[/mm] >0. Sei f: [mm]\IC\to \IC[/mm] ganz und es
> > > > gelte |f(z)| [mm]\le A+B|z|^\alpha[/mm] für alle z [mm]\in \IC.[/mm] Zeigen
> > > > Sie, dass f ein Polynom ist, dessen Grad höchstens [a]
> > > > beträgt.
> > > > b) Bestimmen Sie alle holomorphen Funktionen g: [mm]\IC \setminus[/mm]
> > > > {0} [mm]\to \IC[/mm] mit der Eigenschaft
>  >  >  >  [mm]|g(z)|\le \sqrt{|z|}+\frac{1}{\sqrt{|z|}}[/mm]
>  >  >  
> >  

> > Hallo
> > > :-)
>  >  >  >  
> > > > zu Aufgabe a)
>  >  >  >  ich weiß, dass f holomorph und beschränkt ist,
> > >
> > > Hmmm...., da weißt Du etwas, das gar nicht stimmt !
> > >
> > > Nimm mal f(z)=z
>  >  >  
> > > f ist holomorph auf [mm]\IC,[/mm] das stimmt, aber beschränkt ....
> > > ?
>  >  >  
> > >
> > >
> > >
> > > > aber was
> > > > genau ist A und B? sind das Elemente aus [mm]\IC[/mm] oder
> > > > Funktionen? Die Klammern in denen a steht sind übrigens
> > > > oben nicht geschlossen (dieses Zeichen gab es hier nur
> > > > nicht ). Was bedeutet dieses Zeichen dann?
>  >  >  
> > > Das hat Gono Dir gesagt.
>  >  >  
> > >
> > > >  

> > > > zu Aufgabe b)
>  >  >  >  kann mir jemand einen Tipp geben? Kann man hier
> was
> > mit
> > > > Potenzeihen machen?Holomorphe FUnktionen lassen sich ja
> > > > schliesslich in eine Potenzreihe entwickeln....
>  >  >  
> > > Die Aufgabe hat es in sich...
>  >  >  
> > > Der Vorschlag von Gono ist in meinen Augen nicht brauchbar,
> > > denn multiplizieren wir mit [mm]\wurzel{|z|}[/mm] durch, so bekommen
> > > wir
>  >  >  
> > > [mm]\wurzel{|z|}|g(z)| \le[/mm] 1+|z|  für z [mm]\ne[/mm] 0.
>  >  >  
> > > Wenn wir nun a) anwenden wollen, brauchen wir eine ganze
> > > Funktion f. Aber welch soll das sein ? Es bietet sich
> > > natürlich  [mm]\wurzel{z}g(z)[/mm] an. Das macht aber Probleme,
> > > denn g ist in 0 nicht definiert und welche (holomorphe)
> > > Wurzel nehmen wir .. ?
>  >  >  
> > >
> > > Wir multiplizieren mit z durch und bekommen, mit
> > > h(z):=zg(z):
>  >  >  
> > > (*) [mm]|h(z)| \le z \wurzel{|z|}+ \wurzel{|z|}[/mm]  für z [mm]\ne[/mm] 0.
>  
>
> Ich hatte mich oben verschrieben. Es lautet natürlich
>  
> (*) [mm]|h(z)| \le |z| \wurzel{|z|}+ \wurzel{|z|}[/mm]  für z [mm]\ne[/mm]
> 0.
>  
>
>
> >  

> > Warum?
> > Ich erhalte da für
>  >  [mm]|h(z)|\le\wurzel{|z|}*z+\frac{z}{\wurzel{|z|}}[/mm]
>  
>
> Es soll so lauten (Beträge !)
>  
> [mm]|h(z)|\le\wurzel{|z|}*|z|+\frac{|z|}{\wurzel{|z|}}[/mm]
>  
> Weiter ist [mm]\frac{|z|}{\wurzel{|z|}}=\wurzel{|z|}[/mm]
>  
>
>
> >  >  

> > > h hat in 0 eine isolierte Singularität. Zeige Du, mit (*),
> > > dass diese Singularität hebbar ist.
>  >  Ok ich setzt mich gleich ran ;-)

Da ich mit Singularitäten noch nicht vertraut bin (kam noch nicht in der VL vor) habe ich mir mal ein paar Beispielaufgaben angeguckt und festgestellt, dass oftmals Ableitungen gebildet werden und wenn die beschränkt waren (ich glaube in dem Punkt, wo die Singularität liegt) dann handelte es sich um eine hebbare Singularität. Aber kann man immer so vorgehen? In den BEispielaufgaben die ich mir angeschaut hatte, ging es oftmals um Brüche, wo im Nenner dann die Singularität stand und mit der Regel von Hopital gearbeitet wurde....

>  >  >  
> > > h kann also durch h(0):=0 zu einer ganzen Funktion
> > > fortgesetzt werden.
>  >  >  
> > > Für z [mm]\ne[/mm] 0 ist
> > >
> > > [mm]g(z)=\bruch{h(z)}{z}=\bruch{h(z)-h(0)}{z-0} \to[/mm] h'(0)  
> > > für z [mm]\to[/mm] 0.
>  >  >  
> > > g hat in 0 eine isolierte Singularität. Zeige Du, dass
> > > diese Singularität hebbar ist.
>  >  >  

Wenn ich das richtig verstanden habe, könnte man die ABleitung mittels der Regel von Hopital bilden und man hätte [mm] \frac{h'(z)}{1} [/mm] wobei |h(z)| ja schon beschränkt ist und dann auch h'(z) beschränkt sein muss und damit folgt die hebbarkeit?!

> > > g kann also durch g(0):=h'(0) zu einer ganzen Funktion
> > > fortgesetzt werden.
>  >  >  
> > > g ist dann auf [mm]\{z \in \IC: |z| \le 1 \}[/mm] beschränkt. Es
> > > gibt also ein c>0 mit
>  >  >  
> > > |g(z)| [mm]\le[/mm] c für |z| [mm]\le[/mm] 1,
>  >  >  
> > > also auch
>  >  >  
> > > |g(z)| [mm]\le[/mm] c +|z|  für |z| [mm]\le[/mm] 1.
>  >  >  
> > > Für |z| [mm]\ge[/mm] 1 folgt aus  [mm]|g(z)|\le \sqrt{|z|}+\frac{1}{\sqrt{|z|}} [/mm]:
>  
> >  

> > >  

> > > |g(z)| [mm]\le 1+\sqrt{|z|} \le[/mm] 1+|z|.
>  >  >  
> > > Setzt man A:= [mm]\max\{1,c\},[/mm] so folgt
>  >  >  
> > > |g(z)| [mm]\le[/mm] A+|z|  für alle z [mm]\in \IC.[/mm]
>  >  >  
> > > Jetzt können wir a) anwenden und sehen: es gibt a,b [mm]\in \IC[/mm]
> > > mit:
>  >  >  
> > > g(z)=a+bz.
>  >  >  
> > > Jetzt müssen wir noch genaueres über a und b
> > > rauskitzeln.
>  >  >  
> > >
> > > Aus
> > >
> > > [mm]|a+bz|\le \sqrt{|z|}+\frac{1}{\sqrt{|z|}}[/mm]  für z [mm]\ne[/mm] 0
>  >  >  
> > > folgt
>  >  >  
> > > [mm]|a+bz^2|\le |z|+\frac{1}{|z|}[/mm]  für z [mm]\ne[/mm] 0
>  >  >  
> > > Das liefert
>  >  >  
> > > [mm]|az+bz^3| \le 1+|z|^2[/mm]  für z [mm]\ne[/mm] 0.
>  >  >  
> > > Mit z [mm]\to[/mm] 0 sieht man dann
>  >  >  
> > > [mm]|az+bz^3| \le 1+|z|^2[/mm]  für alle z .
>  >  >  
> > > Aufgabenteil a) liefert nun b=0.
>  >  Das verstehe ich leider noch nicht, warum b=0?
>  
>
> Aus  [mm]|az+bz^3| \le 1+|z|^2[/mm]  für alle z  folgt mit a):
>  
> [mm]az+bz^3[/mm] ist ein Polynom vom Grad [mm]\le[/mm] 2.
>  
> Dann muss b=0 sein
>  
> FRED
>  >  > Das hat zur Folge

>  >  >  
> > > [mm]|a|\le \sqrt{|z|}+\frac{1}{\sqrt{|z|}}[/mm]  für z [mm]\ne[/mm] 0
>  >  >  
> > > oder
>  >  >  
> > > [mm]|a| \le \bruch{1}{r}+r[/mm]  für alle r>0
>  >  >  
> > > Überlege Dir: [mm]\min\{ \bruch{1}{r}+r: r>0\}=2.[/mm]
>  >  >  
> > > Welche Funktionen g sind nun die gesuchten ?
>  >  >  
> > > FRED

also ich suche praktisch Funktionen g die aus |a| bestehen und für die gilt |a| [mm] \le [/mm] 2, oder? Aber wenn das z jetzt gar nicht mehr in der Funktion g vorkommt, dann such ich doch nach Konstanten, dessen Betrag kleiner 2 ist, oder? Was ja dann die Antwort auf deine Frage und die AUfgabenstellung wäre :-)

>  >  >  
> > >
> > > >  

> > > >
> > > > Vielen Dank :-)
> > >  

> >  

>  


Bezug
                                        
Bezug
Grad von Polynom: Antwort
Status: (Antwort) fertig Status 
Datum: 18:56 Sa 18.06.2016
Autor: fred97


> > > > > a) Es seien [mm]A,B,\alpha[/mm] >0. Sei f: [mm]\IC\to \IC[/mm] ganz und es
> > > > > gelte |f(z)| [mm]\le A+B|z|^\alpha[/mm] für alle z [mm]\in \IC.[/mm] Zeigen
> > > > > Sie, dass f ein Polynom ist, dessen Grad höchstens [a]
> > > > > beträgt.
> > > > > b) Bestimmen Sie alle holomorphen Funktionen g: [mm]\IC \setminus[/mm]
> > > > > {0} [mm]\to \IC[/mm] mit der Eigenschaft
>  >  >  >  >  [mm]|g(z)|\le \sqrt{|z|}+\frac{1}{\sqrt{|z|}}[/mm]
>  >  
> >  >  

> > >  

> > > Hallo
> > > > :-)
>  >  >  >  >  
> > > > > zu Aufgabe a)
>  >  >  >  >  ich weiß, dass f holomorph und beschränkt
> ist,
> > > >
> > > > Hmmm...., da weißt Du etwas, das gar nicht stimmt !
> > > >
> > > > Nimm mal f(z)=z
>  >  >  >  
> > > > f ist holomorph auf [mm]\IC,[/mm] das stimmt, aber beschränkt ....
> > > > ?
>  >  >  >  
> > > >
> > > >
> > > >
> > > > > aber was
> > > > > genau ist A und B? sind das Elemente aus [mm]\IC[/mm] oder
> > > > > Funktionen? Die Klammern in denen a steht sind übrigens
> > > > > oben nicht geschlossen (dieses Zeichen gab es hier nur
> > > > > nicht ). Was bedeutet dieses Zeichen dann?
>  >  >  >  
> > > > Das hat Gono Dir gesagt.
>  >  >  >  
> > > >
> > > > >  

> > > > > zu Aufgabe b)
>  >  >  >  >  kann mir jemand einen Tipp geben? Kann man
> hier
> > was
> > > mit
> > > > > Potenzeihen machen?Holomorphe FUnktionen lassen sich ja
> > > > > schliesslich in eine Potenzreihe entwickeln....
>  >  >  >  
> > > > Die Aufgabe hat es in sich...
>  >  >  >  
> > > > Der Vorschlag von Gono ist in meinen Augen nicht brauchbar,
> > > > denn multiplizieren wir mit [mm]\wurzel{|z|}[/mm] durch, so bekommen
> > > > wir
>  >  >  >  
> > > > [mm]\wurzel{|z|}|g(z)| \le[/mm] 1+|z|  für z [mm]\ne[/mm] 0.
>  >  >  >  
> > > > Wenn wir nun a) anwenden wollen, brauchen wir eine ganze
> > > > Funktion f. Aber welch soll das sein ? Es bietet sich
> > > > natürlich  [mm]\wurzel{z}g(z)[/mm] an. Das macht aber Probleme,
> > > > denn g ist in 0 nicht definiert und welche (holomorphe)
> > > > Wurzel nehmen wir .. ?
>  >  >  >  
> > > >
> > > > Wir multiplizieren mit z durch und bekommen, mit
> > > > h(z):=zg(z):
>  >  >  >  
> > > > (*) [mm]|h(z)| \le z \wurzel{|z|}+ \wurzel{|z|}[/mm]  für z [mm]\ne[/mm] 0.
>  >  
> >
> > Ich hatte mich oben verschrieben. Es lautet natürlich
>  >  
> > (*) [mm]|h(z)| \le |z| \wurzel{|z|}+ \wurzel{|z|}[/mm]  für z [mm]\ne[/mm]
> > 0.
>  >  
> >
> >
> > >  

> > > Warum?
> > > Ich erhalte da für
>  >  >  [mm]|h(z)|\le\wurzel{|z|}*z+\frac{z}{\wurzel{|z|}}[/mm]
>  >  
> >
> > Es soll so lauten (Beträge !)
>  >  
> > [mm]|h(z)|\le\wurzel{|z|}*|z|+\frac{|z|}{\wurzel{|z|}}[/mm]
>  >  
> > Weiter ist [mm]\frac{|z|}{\wurzel{|z|}}=\wurzel{|z|}[/mm]
>  >  
> >
> >
> > >  >  

> > > > h hat in 0 eine isolierte Singularität. Zeige Du, mit (*),
> > > > dass diese Singularität hebbar ist.
>  >  >  Ok ich setzt mich gleich ran ;-)
>  Da ich mit Singularitäten noch nicht vertraut bin (kam
> noch nicht in der VL vor) habe ich mir mal ein paar
> Beispielaufgaben angeguckt und festgestellt, dass oftmals
> Ableitungen gebildet werden und wenn die beschränkt waren
> (ich glaube in dem Punkt, wo die Singularität liegt) dann
> handelte es sich um eine hebbare Singularität. Aber kann
> man immer so vorgehen? In den BEispielaufgaben die ich mir
> angeschaut hatte, ging es oftmals um Brüche, wo im Nenner
> dann die Singularität stand und mit der Regel von Hopital
> gearbeitet wurde....
> >  >  >  

> > > > h kann also durch h(0):=0 zu einer ganzen Funktion
> > > > fortgesetzt werden.
>  >  >  >  
> > > > Für z [mm]\ne[/mm] 0 ist
> > > >
> > > > [mm]g(z)=\bruch{h(z)}{z}=\bruch{h(z)-h(0)}{z-0} \to[/mm] h'(0)  
> > > > für z [mm]\to[/mm] 0.
>  >  >  >  
> > > > g hat in 0 eine isolierte Singularität. Zeige Du, dass
> > > > diese Singularität hebbar ist.
>  >  >  >  
> Wenn ich das richtig verstanden habe, könnte man die
> ABleitung mittels der Regel von Hopital bilden


ich hab den Differenzenquotienten benutzt....


>  und man
> hätte [mm]\frac{h'(z)}{1}[/mm] wobei |h(z)| ja schon beschränkt
> ist und dann auch h'(z) beschränkt sein muss und damit
> folgt die hebbarkeit?!


Nein. g ist in der nähe von 0 beschränkt.


> > > > g kann also durch g(0):=h'(0) zu einer ganzen Funktion
> > > > fortgesetzt werden.
>  >  >  >  
> > > > g ist dann auf [mm]\{z \in \IC: |z| \le 1 \}[/mm] beschränkt. Es
> > > > gibt also ein c>0 mit
>  >  >  >  
> > > > |g(z)| [mm]\le[/mm] c für |z| [mm]\le[/mm] 1,
>  >  >  >  
> > > > also auch
>  >  >  >  
> > > > |g(z)| [mm]\le[/mm] c +|z|  für |z| [mm]\le[/mm] 1.
>  >  >  >  
> > > > Für |z| [mm]\ge[/mm] 1 folgt aus  [mm]|g(z)|\le \sqrt{|z|}+\frac{1}{\sqrt{|z|}} [/mm]:
>  
> >  

> > >  

> > > >  

> > > > |g(z)| [mm]\le 1+\sqrt{|z|} \le[/mm] 1+|z|.
>  >  >  >  
> > > > Setzt man A:= [mm]\max\{1,c\},[/mm] so folgt
>  >  >  >  
> > > > |g(z)| [mm]\le[/mm] A+|z|  für alle z [mm]\in \IC.[/mm]
>  >  >  >  
> > > > Jetzt können wir a) anwenden und sehen: es gibt a,b [mm]\in \IC[/mm]
> > > > mit:
>  >  >  >  
> > > > g(z)=a+bz.
>  >  >  >  
> > > > Jetzt müssen wir noch genaueres über a und b
> > > > rauskitzeln.
>  >  >  >  
> > > >
> > > > Aus
> > > >
> > > > [mm]|a+bz|\le \sqrt{|z|}+\frac{1}{\sqrt{|z|}}[/mm]  für z [mm]\ne[/mm] 0
>  >  >  >  
> > > > folgt
>  >  >  >  
> > > > [mm]|a+bz^2|\le |z|+\frac{1}{|z|}[/mm]  für z [mm]\ne[/mm] 0
>  >  >  >  
> > > > Das liefert
>  >  >  >  
> > > > [mm]|az+bz^3| \le 1+|z|^2[/mm]  für z [mm]\ne[/mm] 0.
>  >  >  >  
> > > > Mit z [mm]\to[/mm] 0 sieht man dann
>  >  >  >  
> > > > [mm]|az+bz^3| \le 1+|z|^2[/mm]  für alle z .
>  >  >  >  
> > > > Aufgabenteil a) liefert nun b=0.
>  >  >  Das verstehe ich leider noch nicht, warum b=0?
>  >  
> >
> > Aus  [mm]|az+bz^3| \le 1+|z|^2[/mm]  für alle z  folgt mit a):
>  >  
> > [mm]az+bz^3[/mm] ist ein Polynom vom Grad [mm]\le[/mm] 2.
>  >  
> > Dann muss b=0 sein
>  >  
> > FRED
>  >  >  > Das hat zur Folge

>  >  >  >  
> > > > [mm]|a|\le \sqrt{|z|}+\frac{1}{\sqrt{|z|}}[/mm]  für z [mm]\ne[/mm] 0
>  >  >  >  
> > > > oder
>  >  >  >  
> > > > [mm]|a| \le \bruch{1}{r}+r[/mm]  für alle r>0
>  >  >  >  
> > > > Überlege Dir: [mm]\min\{ \bruch{1}{r}+r: r>0\}=2.[/mm]
>  >  >  
> >  

> > > > Welche Funktionen g sind nun die gesuchten ?
>  >  >  >  
> > > > FRED
>  also ich suche praktisch Funktionen g die aus |a| bestehen
> und für die gilt |a| [mm]\le[/mm] 2, oder?


Ja

> Aber wenn das z jetzt
> gar nicht mehr in der Funktion g vorkommt, dann such ich
> doch nach Konstanten, dessen Betrag kleiner 2 ist, oder?


Ja



> Was ja dann die Antwort auf deine Frage und die
> AUfgabenstellung wäre :-)

Ja


FRED

>  >  >  >  
> > > >
> > > > >  

> > > > >
> > > > > Vielen Dank :-)
> > > >  

> > >  

> >  

>  


Bezug
                                                
Bezug
Grad von Polynom: Frage (überfällig)
Status: (Frage) überfällig Status 
Datum: 19:17 Sa 18.06.2016
Autor: Herzblatt


> > > > > > a) Es seien [mm]A,B,\alpha[/mm] >0. Sei f: [mm]\IC\to \IC[/mm] ganz und es
> > > > > > gelte |f(z)| [mm]\le A+B|z|^\alpha[/mm] für alle z [mm]\in \IC.[/mm] Zeigen
> > > > > > Sie, dass f ein Polynom ist, dessen Grad höchstens [a]
> > > > > > beträgt.
> > > > > > b) Bestimmen Sie alle holomorphen Funktionen g: [mm]\IC \setminus[/mm]
> > > > > > {0} [mm]\to \IC[/mm] mit der Eigenschaft
>  >  >  >  >  >  [mm]|g(z)|\le \sqrt{|z|}+\frac{1}{\sqrt{|z|}}[/mm]
>  
> >  >  

> > >  >  

> > > >  

> > > > Hallo
> > > > > :-)
>  >  >  >  >  >  
> > > > > > zu Aufgabe a)
>  >  >  >  >  >  ich weiß, dass f holomorph und beschränkt
> > ist,
> > > > >
> > > > > Hmmm...., da weißt Du etwas, das gar nicht stimmt !
> > > > >
> > > > > Nimm mal f(z)=z
>  >  >  >  >  
> > > > > f ist holomorph auf [mm]\IC,[/mm] das stimmt, aber beschränkt ....
> > > > > ?
>  >  >  >  >  
> > > > >
> > > > >
> > > > >
> > > > > > aber was
> > > > > > genau ist A und B? sind das Elemente aus [mm]\IC[/mm] oder
> > > > > > Funktionen? Die Klammern in denen a steht sind übrigens
> > > > > > oben nicht geschlossen (dieses Zeichen gab es hier nur
> > > > > > nicht ). Was bedeutet dieses Zeichen dann?
>  >  >  >  >  
> > > > > Das hat Gono Dir gesagt.
>  >  >  >  >  
> > > > >
> > > > > >  

> > > > > > zu Aufgabe b)
>  >  >  >  >  >  kann mir jemand einen Tipp geben? Kann man
> > hier
> > > was
> > > > mit
> > > > > > Potenzeihen machen?Holomorphe FUnktionen lassen sich ja
> > > > > > schliesslich in eine Potenzreihe entwickeln....
>  >  >  >  >  
> > > > > Die Aufgabe hat es in sich...
>  >  >  >  >  
> > > > > Der Vorschlag von Gono ist in meinen Augen nicht brauchbar,
> > > > > denn multiplizieren wir mit [mm]\wurzel{|z|}[/mm] durch, so bekommen
> > > > > wir
>  >  >  >  >  
> > > > > [mm]\wurzel{|z|}|g(z)| \le[/mm] 1+|z|  für z [mm]\ne[/mm] 0.
>  >  >  >  >  
> > > > > Wenn wir nun a) anwenden wollen, brauchen wir eine ganze
> > > > > Funktion f. Aber welch soll das sein ? Es bietet sich
> > > > > natürlich  [mm]\wurzel{z}g(z)[/mm] an. Das macht aber Probleme,
> > > > > denn g ist in 0 nicht definiert und welche (holomorphe)
> > > > > Wurzel nehmen wir .. ?
>  >  >  >  >  
> > > > >
> > > > > Wir multiplizieren mit z durch und bekommen, mit
> > > > > h(z):=zg(z):
>  >  >  >  >  
> > > > > (*) [mm]|h(z)| \le z \wurzel{|z|}+ \wurzel{|z|}[/mm]  für z [mm]\ne[/mm] 0.
>  >  >  
> > >
> > > Ich hatte mich oben verschrieben. Es lautet natürlich
>  >  >  
> > > (*) [mm]|h(z)| \le |z| \wurzel{|z|}+ \wurzel{|z|}[/mm]  für z [mm]\ne[/mm]
> > > 0.
>  >  >  
> > >
> > >
> > > >  

> > > > Warum?
> > > > Ich erhalte da für
>  >  >  >  [mm]|h(z)|\le\wurzel{|z|}*z+\frac{z}{\wurzel{|z|}}[/mm]
>  >  >  
> > >
> > > Es soll so lauten (Beträge !)
>  >  >  
> > > [mm]|h(z)|\le\wurzel{|z|}*|z|+\frac{|z|}{\wurzel{|z|}}[/mm]
>  >  >  
> > > Weiter ist [mm]\frac{|z|}{\wurzel{|z|}}=\wurzel{|z|}[/mm]
>  >  >  
> > >
> > >
> > > >  >  

> > > > > h hat in 0 eine isolierte Singularität. Zeige Du, mit (*),
> > > > > dass diese Singularität hebbar ist.
>  >  >  >  Ok ich setzt mich gleich ran ;-)
>  >  Da ich mit Singularitäten noch nicht vertraut bin (kam
> > noch nicht in der VL vor) habe ich mir mal ein paar
> > Beispielaufgaben angeguckt und festgestellt, dass oftmals
> > Ableitungen gebildet werden und wenn die beschränkt waren
> > (ich glaube in dem Punkt, wo die Singularität liegt) dann
> > handelte es sich um eine hebbare Singularität. Aber kann
> > man immer so vorgehen? In den BEispielaufgaben die ich mir
> > angeschaut hatte, ging es oftmals um Brüche, wo im Nenner
> > dann die Singularität stand und mit der Regel von Hopital
> > gearbeitet wurde....
> > >  >  >  

> > > > > h kann also durch h(0):=0 zu einer ganzen Funktion
> > > > > fortgesetzt werden.
>  >  >  >  >  
> > > > > Für z [mm]\ne[/mm] 0 ist
> > > > >
> > > > > [mm]g(z)=\bruch{h(z)}{z}=\bruch{h(z)-h(0)}{z-0} \to[/mm] h'(0)  
> > > > > für z [mm]\to[/mm] 0.
>  >  >  >  >  
> > > > > g hat in 0 eine isolierte Singularität. Zeige Du, dass
> > > > > diese Singularität hebbar ist.
>  >  >  >  >  
> > Wenn ich das richtig verstanden habe, könnte man die
> > ABleitung mittels der Regel von Hopital bilden
>  
>
> ich hab den Differenzenquotienten benutzt....
>  
>
> >  und man

> > hätte [mm]\frac{h'(z)}{1}[/mm] wobei |h(z)| ja schon beschränkt
> > ist und dann auch h'(z) beschränkt sein muss und damit
> > folgt die hebbarkeit?!
>
>
> Nein. g ist in der nähe von 0 beschränkt.
>  
>

Kann ich das untenstehende trotzdem mit L'Hopital folgern?

[mm] g(z)=\frac{h(z)}{z} [/mm] und mit L'Hopital gilt für  [mm] \frac{h'(z)}{z'}=\frac{g'(z)*z+g(z)}{1} [/mm] und nun an der Singularität 0 gilt [mm] \frac{g'(0)*0+g(0)}{1}=\frac{0}{1}=0 [/mm] ist und die Singularität hebbar

> > > > > g kann also durch g(0):=h'(0) zu einer ganzen Funktion
> > > > > fortgesetzt werden.
>  >  >  >  >  
> > > > > g ist dann auf [mm]\{z \in \IC: |z| \le 1 \}[/mm] beschränkt. Es
> > > > > gibt also ein c>0 mit
>  >  >  >  >  
> > > > > |g(z)| [mm]\le[/mm] c für |z| [mm]\le[/mm] 1,
>  >  >  >  >  
> > > > > also auch
>  >  >  >  >  
> > > > > |g(z)| [mm]\le[/mm] c +|z|  für |z| [mm]\le[/mm] 1.
>  >  >  >  >  
> > > > > Für |z| [mm]\ge[/mm] 1 folgt aus  [mm]|g(z)|\le \sqrt{|z|}+\frac{1}{\sqrt{|z|}} [/mm]:
>  
> >  

> > >  

> > > >  

> > > > >  

> > > > > |g(z)| [mm]\le 1+\sqrt{|z|} \le[/mm] 1+|z|.
>  >  >  >  >  
> > > > > Setzt man A:= [mm]\max\{1,c\},[/mm] so folgt
>  >  >  >  >  
> > > > > |g(z)| [mm]\le[/mm] A+|z|  für alle z [mm]\in \IC.[/mm]
>  >  >  >  >  
> > > > > Jetzt können wir a) anwenden und sehen: es gibt a,b [mm]\in \IC[/mm]
> > > > > mit:
>  >  >  >  >  
> > > > > g(z)=a+bz.
>  >  >  >  >  
> > > > > Jetzt müssen wir noch genaueres über a und b
> > > > > rauskitzeln.
>  >  >  >  >  
> > > > >
> > > > > Aus
> > > > >
> > > > > [mm]|a+bz|\le \sqrt{|z|}+\frac{1}{\sqrt{|z|}}[/mm]  für z [mm]\ne[/mm] 0
>  >  >  >  >  
> > > > > folgt
>  >  >  >  >  
> > > > > [mm]|a+bz^2|\le |z|+\frac{1}{|z|}[/mm]  für z [mm]\ne[/mm] 0
>  >  >  >  >  
> > > > > Das liefert
>  >  >  >  >  
> > > > > [mm]|az+bz^3| \le 1+|z|^2[/mm]  für z [mm]\ne[/mm] 0.
>  >  >  >  >  
> > > > > Mit z [mm]\to[/mm] 0 sieht man dann
>  >  >  >  >  
> > > > > [mm]|az+bz^3| \le 1+|z|^2[/mm]  für alle z .
>  >  >  >  >  
> > > > > Aufgabenteil a) liefert nun b=0.
>  >  >  >  Das verstehe ich leider noch nicht, warum b=0?
>  >  >  
> > >
> > > Aus  [mm]|az+bz^3| \le 1+|z|^2[/mm]  für alle z  folgt mit a):
>  >  >  
> > > [mm]az+bz^3[/mm] ist ein Polynom vom Grad [mm]\le[/mm] 2.
>  >  >  
> > > Dann muss b=0 sein
>  >  >  
> > > FRED
>  >  >  >  > Das hat zur Folge

>  >  >  >  >  
> > > > > [mm]|a|\le \sqrt{|z|}+\frac{1}{\sqrt{|z|}}[/mm]  für z [mm]\ne[/mm] 0
>  >  >  >  >  
> > > > > oder
>  >  >  >  >  
> > > > > [mm]|a| \le \bruch{1}{r}+r[/mm]  für alle r>0
>  >  >  >  >  
> > > > > Überlege Dir: [mm]\min\{ \bruch{1}{r}+r: r>0\}=2.[/mm]
>  >  
> >  >  

> > >  

> > > > > Welche Funktionen g sind nun die gesuchten ?
>  >  >  >  >  
> > > > > FRED
>  >  also ich suche praktisch Funktionen g die aus |a|
> bestehen
> > und für die gilt |a| [mm]\le[/mm] 2, oder?
>
>
> Ja
>  
> > Aber wenn das z jetzt
> > gar nicht mehr in der Funktion g vorkommt, dann such ich
> > doch nach Konstanten, dessen Betrag kleiner 2 ist, oder?
>
>
> Ja
>  
>
>
> > Was ja dann die Antwort auf deine Frage und die
> > AUfgabenstellung wäre :-)
>  
> Ja
>  
>
> FRED
>  >  >  >  >  
> > > > >
> > > > > >  

> > > > > >
> > > > > > Vielen Dank :-)
> > > > >  

> > > >  

> > >  

> >  

>  


Bezug
                                                        
Bezug
Grad von Polynom: Fälligkeit abgelaufen
Status: (Mitteilung) Reaktion unnötig Status 
Datum: 19:20 Mo 20.06.2016
Autor: matux

$MATUXTEXT(ueberfaellige_frage)
Bezug
Ansicht: [ geschachtelt ] | ^ Forum "Uni-Komplexe Analysis"  | ^^ Alle Foren  | ^ Forenbaum  | Materialien


^ Seitenanfang ^
www.vorhilfe.de